2014 dxdy logo

Научный форум dxdy

Математика, Физика, Computer Science, Machine Learning, LaTeX, Механика и Техника, Химия,
Биология и Медицина, Экономика и Финансовая Математика, Гуманитарные науки




Начать новую тему Ответить на тему На страницу Пред.  1, 2, 3, 4, 5 ... 7  След.
 
 Re: Задача с треугольными числами
Сообщение23.09.2023, 16:35 
Заслуженный участник
Аватара пользователя


21/11/12
1881
Санкт-Петербург
scwec в сообщении #1610968 писал(а):
Для полноты картины рассмотрим $R=1$
Нехарактерный случай. Ему соответствует бесконечная серия пифагоровых троек, полученных из последовательности $1+\sqrt{2}=2,2,2,...=\dfrac{2}{1},\dfrac{5}{2},\dfrac{12}{5},...$ Все они сокращаются в единицу. Отсюда же видно, что поиски ограничений сократимости будут затруднительны.

 Профиль  
                  
 
 Re: Задача с треугольными числами
Сообщение23.09.2023, 20:24 
Заслуженный участник


17/09/10
2133
Ну, "для полноты картины" имелось в виду, что для $R=1$ разобранная выше эллиптическая кривая не действует.
Теперь все параметрические решения для $R=1$ выложены. Так что картина классификации положительных $R$ в этом смысле полная.

 Профиль  
                  
 
 Re: Задача с треугольными числами
Сообщение04.10.2023, 13:51 
Заслуженный участник


17/09/10
2133
Одно замечание к случаю $R=2$
Решая квадратное уравнение $u^2-4uv+v^2-1=0$ находим
$u=2v\pm\sqrt{3v^2+1}$
$v=2u\pm\sqrt{3u^2+1}$
$u,v$ являются соседними членами в последовательности $\{y_n\}$ OEIS A001353 решений уравнения Пелля $x^2-3y^2=1$.
Вывод. $y_n^2+y_{n+1}^2$ не может быть полным квадратом.
Это следует из доказанного (c помощью разобранной выше эллиптической кривой) утверждения, что для $R=2$ уравнение $\dfrac{w^2-1}{2uv}=R$, где $u^2+v^2=w^2$, не имеет решения в натуральных (и даже в рациональных) числах $u,v,w$.
Такой подход можно применить и для любого натурального $R$.
В данном случае мы имеем дело с уравнение Пелля $x^2-(R^2-1)y^2=1$. Тут решения исходного уравнения не исключаются и искать решения $u,v$ нужно среди соседних $y_n,y_{n+1}$ таких, что $y_n^2+y_{n+1}^2$ является квадратом.
Раcчеты упрощаются тем, что надо просто последовательно перебирать пары.
Закон следования простой $y_{n+1}=2Ry_n-y_{n-1}$, а первые члены $y_1=1,y_2=2R$.
Ну, а для $R=2$ перебирать ничего не надо.

 Профиль  
                  
 
 Re: Задача с треугольными числами
Сообщение05.10.2023, 12:13 
Заслуженный участник
Аватара пользователя


21/11/12
1881
Санкт-Петербург
scwec в сообщении #1612394 писал(а):
Такой подход можно применить и для любого натурального $R$.
Да. Но $R$ у нас несократимая дробь. Я конечно рассматривал уравнение $u^2+v^2-2Ruv-1=0$ и два Пелля, следующих из него симметрично:
$(u-vR)^2-(R^2-1)v^2=1.$
$(v-uR)^2-(R^2-1)u^2=1.$
Чтобы старший квадрат оказался целым числом, надо домножить всё на $q^2,$ т.е. к классическому Пеллю не свести. Преимущество же уравнения $\dfrac{x^2-1}{y^2-1}=m\ (3)$ в том, что решение следует из одной дроби, которая может оказаться и сократимой (это важно). Остальное всё как у Вас:
scwec в сообщении #1612394 писал(а):
... надо просто последовательно перебирать пары.
И только сейчас обратил внимание: если существуют решения в больших номерах, должно хорошо выполняться $\sqrt{R^2-1} \approx \dfrac{u}{v}-R,$ откуда $\dfrac{u}{v} \approx R+\sqrt{R^2-1},$ т.е. отношение искомых переменных определено довольно точно. Отсюда жесткость условий разрешимости, но и возможность новых подходов. Об этом пока не готов. Надо поближе подпустить )

 Профиль  
                  
 
 Re: Задача с треугольными числами
Сообщение08.10.2023, 06:16 
Заслуженный участник
Аватара пользователя


21/11/12
1881
Санкт-Петербург
Andrey A в сообщении #1612532 писал(а):
... т.е. отношение искомых переменных определено довольно точно.
Дробь $\dfrac{u}{v}$ — вовсе не любая рациональная точка (отношение младших элементов пифагоровой тройки), но любая рациональная точка задает такую дробь* (функция от рациональной переменной): \dfrac{u}{v}=\dfrac{a^2-b^2}{2ab}=\dfrac{h^2-1}{2h},$ где $h=\dfrac{a}{b}.$ Вот и попробуем свести неопределенности к минимуму, приравняв $\dfrac{h^2-1}{2h} \approx R+\sqrt{R^2-1}.$ Имеем уравнение $4$-й степени $h^4-4Rh^3+2h^2+4Rh+1 \approx 0.$ Подставляя $R=\dfrac{p}{q}>1,$ получаем уравнение с целыми коэффициентами: $$qh^4-4ph^3+2qh^2+4ph+q \approx 0\ \ (4),$$ которое многое проясняет. Приближенные значения корня $(4)\ h_n$ задают примитивные тройки нужной пропорции, но искомые пифагоровы тройки не обязаны быть примитивными (из условия). Положим $\gcd (u_n,v_n)=d$ и запишем условие так: $\dfrac{w_n^2d^2-1}{2u_nv_nd^2}=\dfrac{p}{q}=R.$ Отсюда $d^2=\dfrac{1}{w_n^2-2Ru_nv_n}=\dfrac{q}{qw_n^2-2pu_nv_n=\Delta_n}$ и $$d=\sqrt{\dfrac{q}{\Delta_n}}\ \ (5).$$ Чтобы радикал оказался рациональным числом, достаточно $$q\Delta_n=\square\ \ (6)$$ (формула пошагового тестирования). Повторюсь, точные решения $(4)$ нам не нужны, даже если они есть. A нужны приближенные значения $h_n,$ из которых можно получить (по предположению) примитивные тройки, выдерживающие тест формулой $(6)$. Ну, а способ аппроксимации алгебраического числа описан здесь. Пусть это будет самореклама. Очередное разложение. Да. Но по первому впечатлению такой поиск оказывается гораздо более "зрячий", поскольку остатки $\Delta_n$ быстро растут (как и положено для разложения $4$-й степени), и возможность "обратного скачка" связана с очень большим последующим знаком. Этим же объясняется отсутствие решений в больших номерах, и становится очевидной неразрешимость задачи для большинства $R$ при случайном выборе. Несколько разложений для примера.

$R=\dfrac{7}{3}:$ $\begin{matrix}
n & | &\ 1 & 2 & 3 & 4 & 5 & 6 & 7 & 8\\ 
---& + &---&---&---&---&---&---&---&---\\ 
a_n& | & 1,& 3,& 1,& 344,& 1,& 65,& 1,& ...\\
 & | &  &  &  &  &  &  &  & \\ 
\dfrac{p_n}{q_n} & | & \dfrac{1}{1} & \dfrac{4}{3} & \dfrac{5}{4} & \dfrac{1724}{1379} & \dfrac{1729}{1383} & ... &  & \\ 
 ---& + &---&---&---&---&---&---&---&---\\ 
u_n & | & 1 & 24 & 40 & 4754792 & 2391207 & ... &  & \\ 
v_n & | & 0 & 7 & 9 & 1070535 & 538376 & ... &  & \\ 
\Delta_n & | & 3 & -477 & 3 & -121103613 & 465027 & ... &  & 
\end{matrix}$

$d=\sqrt{\dfrac{q}{\Delta_3}}=\sqrt{\dfrac{3}{3}}=1.\ u=u_3 \cdot 1=40,v=v_3 \cdot 1=9,w=\sqrt{40^2+9^2}=41.$

Тут вторая строка — знаки непрерывной дроби разложения корня $(4),$ третья — соотв. подходящие дроби.


$R=\dfrac{15}{13}:$ $\begin{matrix}
n & | &\ 1 & 2 & 3 & 4 & 5 & 6 & 7 & 8\\ 
---& + &---&---&---&---&---&---&---&---\\ 
a_n& | & 1,& 1,& 2,& 1,& 3,& 2074,& 2,& ...\\
 & | &  &  &  &  &  &  &  & \\ 
\dfrac{p_n}{q_n} & | & \dfrac{1}{1} & \dfrac{2}{1} & \dfrac{5}{3} & \dfrac{7}{4} & \dfrac{26}{15} & \dfrac{53931}{31114} & ... & \\ 
 ---& + &---&---&---&---&---&---&---&---\\ 
u_n & | & 1 & 4 & 15 & 56 & 780 & 3356018268 & ... & \\ 
v_n & | & 0 & 3 & 8 & 33 & 451 & 1940471765 & ... & \\ 
\Delta_n & | & 13 & -35 &157 & -515 & 13 & ... &  & 
\end{matrix}$

$d=\sqrt{\dfrac{q}{\Delta_5}}=\sqrt{\dfrac{13}{13}}=1.\ u=780,v=451,w=\sqrt{780^2+451^2}=901.$


$R=\dfrac{3884}{3125}:$ $\begin{matrix}
n & | &\ 1 & 2 & 3 & 4 & 5 & 6 & 7 & 8\\ 
---& + &---&---&---&---&---&---&---&---\\ 
a_n& | & 1,& 1,& 1,& 1,& 2,& 429926,& 1,& ...\\
 & | &  &  &  &  &  &  &  & \\ 
\dfrac{p_n}{q_n} & | & \dfrac{1}{1} & \dfrac{2}{1} & \dfrac{3}{2} & \dfrac{5}{3} & \dfrac{13}{8} & \dfrac{5589043}{3439411} & ... & \\ 
 ---& + &---&---&---&---&---&---&---&---\\ 
u_n & | & 1 & 4 & 12 & 15 & 208 & ... & ... & \\ 
v_n & | & 0 & 3 & 5 & 8 & 105 & ... & ... & \\ 
\Delta_n & | & 3125 & -15091 & 62045 & -29035 & 5 & ... &  & 
\end{matrix}$

Тест: $\sqrt{q\Delta_5}=\sqrt{3125 \cdot 5}=125.$
$d=\sqrt{\dfrac{q}{\Delta_5}}=\sqrt{\dfrac{3125}{5}}=25.\ u=208 \cdot 25=5200,v=105 \cdot 25=2625,$ $w=\sqrt{5200^2+2625^2}=5825.$

Параметр $d$ может оказаться рациональным числом, но не целым; с ростом величин часто так и происходит. Что ж, получаем тогда некоторые рациональные решения — не все, конечно, но "наименьшие".

$R=\dfrac{88}{27}:$ $\begin{matrix}
n & | &\ 1 & 2 & 3 & 4 & 5 & 6 & 7 & 8\\ 
---& + &---&---&---&---&---&---&---&---\\ 
a_n& | & 1,& 5,& 1,& 9,& 26,& 3,&5,& ...\\
 & | &  &  &  &  &  &  &  & \\ 
\dfrac{p_n}{q_n} & | & \dfrac{1}{1} & \dfrac{6}{5} & \dfrac{7}{6} & \dfrac{69}{59} & \dfrac{1801}{1540} & \dfrac{5472}{4679} & ... & \\ 
 ---& + &---&---&---&---&---&---&---&---\\ 
u_n & | & 1 & 60 & 84 & 4071 & 5547080 & ... & ... & \\ 
v_n & | & 0 & 11 & 13 & 640 & 872001 & ... & ... & \\ 
\Delta_n & | & 27 & -15693 & 2883 & -26133 & ... & ... &  & 
\end{matrix}$

Тест: $\sqrt{q\Delta_3}=\sqrt{27 \cdot 2883}=279.$
$d=\sqrt{\dfrac{q}{\Delta_3}}=\sqrt{\dfrac{27}{2883}}=\dfrac{3}{31}.\ u=84 \cdot \dfrac{3}{31}=\dfrac{252}{31},v=13 \cdot \dfrac{3}{31}=\dfrac{39}{31},$ $w=\sqrt{\left ( \dfrac{252}{31} \right )^2+\left ( \dfrac{39}{31} \right )^2}=\dfrac{255}{31}.$

Общее впечатление такое, что исследуя случайные $R$, большей частью будем получать "ровную" дробь и отсутствие решений (что несколько удивительно), реже — рациональные решения (тут зависимость от количества целых делителей $q$) и совсем редко — целые решения. Последнее сопровождается "огромным" знаком цепной дроби и выявляет некое "очень точное" рациональное решение уравнения $(4).$ Любые ли решения находятся таким способом — не знаю. Не уверен. Хвастаться пока нечем. Но интересно, какие еще задачи сводятся к разложению алгебраического числа? Похоже, тут непаханое поле.


*Задает, но двояко. Дело в том, что несократимые дроби $\dfrac{a}{b}$ и $\dfrac{a+b}{a-b}.$ порождают одну и ту же пифагорову тройку.

 Профиль  
                  
 
 Re: Задача с треугольными числами
Сообщение09.10.2023, 04:29 
Заслуженный участник
Аватара пользователя


21/11/12
1881
Санкт-Петербург
Andrey A в сообщении #1612931 писал(а):
... вторая строка — знаки непрерывной дроби разложения корня $(4),$ третья — соотв. подходящие дроби.
... которые и есть $h_n=\dfrac{p_n}{q_n}.$ Главное сказать почему-то забыл. Надо было вывести их большими буквами, чтобы не путалось с $\dfrac{p}{q}=R.$
Andrey A в сообщении #1612931 писал(а):
Любые ли решения находятся таким способом — не знаю.
Имеются в виду целые решения, конечно. Надеюсь, таких пробелов конечное число при "малых" $p,q.$

 Профиль  
                  
 
 Re: Задача с треугольными числами
Сообщение16.10.2023, 14:29 
Заслуженный участник


17/09/10
2133
Докажем, что при $R=3$ исходное уравнение $\dfrac{w^2-1}{2uv}=R$, где $u^2+v^2=w^2$
не имеет решений в натуральных $u,v,w$.
Уже отмечено ранее, что $u=3v\pm{\sqrt{8v^2+1}}$ и $v=3u\pm{\sqrt{8u^2+1}}$
Т,о, $u,v$ - соседние члены последовательности ${\{y_n\}}$ решений уравнения
$x^2-8y^2=1$.
Докажем, что $y_n^2+y_{n-1}^2$ не является квадратом при любом $n$.
Для этого рассмотрим последовательность $\{{y_n^2}\}$.Она содержит все треугольные числа, являющиеся квадратами и никаких других чисел.
Рекуррентное соотношение для неё $y_{n-1}^2{y_n^2}=(y_n^2-1)^2$.
Отсюда $y_{n-1}y_{n+1}+1=y_n^2\qquad(1)$
Для последовательности $\{y_n\}$ справедливы равенства
$y_{n+1}=3y_n+x_n, y_{n-1}=3y_n-x_n$
Откуда $y_{n+1}+y_{n-1}=6y_n\qquad(2)$
Учитывая $(1)$ и $(2)$
$y_n^2+y_{n-1}^2=y_{n-1}y_{n+1}+1+y_{n-1}^2=y_{n-1}(y_{n+1}+y_{n-1})+1=$
$6y_{n-1}{y_n+1}\qquad(3)$
Предположим противное, нашлась пара рядом стоящих квадратов, сумма которых квадрат; $y_n^2+y_{n-1}^2=6y_{n-1}y_n+1=A^2$, где $A$ натуральное число.
тогда $(A^2-1)^2+(6y_n^2)^2=(6y_n{A})^2\qquad(4)$
Стало быть существуют натуральные $M,N,k$ такие, что:
первый вариант: $A^2-1=k(M^2-N^2), 6y_n^2=2kMN, 6Ay_n=k(M^2+N^2)$,
второй вариант: $A^2-1=2kMN, 6y_n^2=k(M^2-N^2), 6Ay_n=k(M^2+N^2)$
В первом варианте при $M>N$
$k=\dfrac{12MN}{M^4 - 12M^3{N} + 2M^2{N^2} + 12MN^3 + N^4}$ (Maple)
Во втором варианте при $M>N$
$k=\dfrac{6(M^2 - N^2)}{M^4 - 12M^3{N} + 2M^2{N^2} + 12MN^3 + N^4}$ (Maple)
Одновременно все $M,N,k$ не могут быть натуральными числами, поскольку
в обоих случаях $k< 1$ при натуральных $M>N$,
Следовательно, тройки нужных натуральных $M,N,k$ не существует, что и доказывает утверждение.
Замечу также ещё раз, что натуральные $u^2,v^2$ такие, что $\dfrac{u^2+v^2}{2uv}=3$ являются треугольными числами. Треугольным является также и число $uv$ (но не квадрат) и три числа $u^2,uv,v^2$ ($u<v$) образуют геометрическую прогрессию.

 Профиль  
                  
 
 Re: Задача с треугольными числами
Сообщение16.10.2023, 21:45 
Заслуженный участник
Аватара пользователя


21/11/12
1881
Санкт-Петербург
scwec
Если хочется свести дело к классическому Пеллю, предложу более общий расклад. Хорошо, пусть $ R$ — целое, $R^2-1=m$ (маленькое $m$ пока свободно) и $x_1^2-my_1^2=1$ — первое решение Пелля.
Остальные решения выражены явно:
$x_n=\dfrac{\alpha^n+\beta^n}{2},$
$y_n=\dfrac{\alpha^n-\beta^n}{2\sqrt{m}},$
где $\alpha=x_1+\sqrt{x_1^2-1},\beta=x_1-\sqrt{x_1^2-1}.$ Сразу заметим, что $\alpha \beta =1.$ Кроме того $x_{n+1}=2x_1x_n-x_{n-1},\ y_{n+1}=2x_1y_n-y_{n-1}\ (x_0=1,y_0=0,y_1=1).$
Тогда
$y_n^2=\left ( \dfrac{\alpha^n-\beta^n}{2 \sqrt{m}} \right )^2=\dfrac{\alpha^{2n}+\beta^{2n}-2 \alpha^n \beta^n}{4m}=$ $\dfrac{x_{2n}-\alpha^n \beta^n}{2m}=\dfrac{x_{2n}-1}{2m}.$

$y_n^2+y_{n-1}^2=\dfrac{x_{2n}+x_{2n-2}-2}{2m}=\dfrac{2x_1x_{2n-1}-2}{2m}=\dfrac{x_1x_{2n-1}-1}{m}.$

В итоге: $$\dfrac{x_1x_{2n-1}-1}{m}=\square$$ Целое оно всегда, но может ли быть квадратом — загадка. Если что-то отсюда следует, можно попробовать продолжить на общий случай.

 Профиль  
                  
 
 Re: Задача с треугольными числами
Сообщение16.10.2023, 23:14 
Заслуженный участник


17/09/10
2133
nnosipov в личном сообщении указал на недостатки в доказательстве утверждения из предыдущего моего текста. А я уже подготовил сообщение с доказательством общего случая натурального $R$. Пока подожду и подумаю.
Кроме этого
scwec в сообщении #1613571 писал(а):
Замечу также ещё раз, что натуральные $u^2,v^2$ такие, что $\dfrac{u^2+v^2}{2uv}=3$

Здесь пропущена $-1$ в записи уравнения. Нужно $\dfrac{u^2+v^2-1}{2uv}=3$

 Профиль  
                  
 
 Re: Задача с треугольными числами
Сообщение17.10.2023, 07:45 
Заслуженный участник
Аватара пользователя


21/11/12
1881
Санкт-Петербург
Andrey A в сообщении #1613622 писал(а):
... $(x_0=1,y_0=0,y_1=1).$
У меня тут тоже ошибка: $y_1 \neq 1$. Нулевая дробь $\dfrac{x_0}{y_0}=\dfrac{1}{0},$ и только.

 Профиль  
                  
 
 Re: Задача с треугольными числами
Сообщение18.10.2023, 09:01 
Заслуженный участник
Аватара пользователя


21/11/12
1881
Санкт-Петербург
Не снимая вопроса, хорошо бы подытожить на школьном уровне: для тройки членов арифметической прогрессии $p-q,p,p+q$ как найти пропорциональную тройку треугольных чисел? В "распакованном" виде уравнение $(3)$ выглядит так: $(p-q)X^2-(p+q)Y^2+2q=0$ или $\left( \dfrac{p-q}{2} \right)X^2-\left( \dfrac{p+q}{2} \right)Y^2+q=0$ (в случае нечетных $p,q$). Заходим на сервис https://www.alpertron.com.ar/QUAD.HTM, выписываем наименьшие неединичные решения этого уравнения $X>Y>0$ и проверяем, нет ли среди них таких, что $\sqrt{\dfrac{X^2+Y^2}{2}}$ — целое число (тест). Назначаем его $w$ (если оно есть), тогда $\dfrac{Y^2-1}{p-q}=\dfrac{w^2-1}{p}=\dfrac{X^2-1}{p+q}$, номера треугольных чисел находим по формулам $(Y-1)/2,(w-1)/2,(X-1)/2.$ В противном случае решений нет, или не могут быть получены школьными методами, что, конечно, не одно и тоже. Ибо не доказано, что другие члены бесконечной последовательности решений $(3)\ \dfrac{X_n}{Y_n}$ не дадут положительного теста, но маловероятно. Знаем только, что случай $R=2/1$ неразрешим, доказал scwec. О причинах такой "маловероятности" говорилось ранее, и есть еще одна. Уравнение $(3)$ в изначальном виде: $\dfrac{R+1}{R-1}=\dfrac{(u+v)^2-1}{(u-v)^2-1}.$ Требуется найти $\dfrac{X_n}{Y_n}=\dfrac{u+v}{u-v}=\dfrac{a^2-b^2+2ab}{a^2-b^2-2ab}=\dfrac{(a+b)^2-2b^2}{(a-b)^2-2b^2}.$ Числитель и знаменатель такой дроби в сокращенном виде не могут иметь в своем каноническом разложении простого делителя вида $8N\pm 3$, но членам последовательности $\dfrac{X_n}{Y_n}$ об этом ничего не известно. С ростом величин вероятность такого состояния падает резко, хотя и в начальных номерах многие решения могут быть отсеяны по этому признаку. О чем и хотелось доложить.

 Профиль  
                  
 
 Re: Задача с треугольными числами
Сообщение18.10.2023, 21:50 
Заслуженный участник


17/09/10
2133
Вношу изменения в доказательство утверждения для $R=3$ (заодно меняем $M,N$ на $p,q$).
После предположения, что $y_n^2+y_{n-1}^2=A^2$ с натуральным $A$
получено уравнение $(A^2-1)^2+(6y^2)^2=(6y{A})^2\qquad(4)$, где $y=y_n$ или $y=y_{n-1}$
Если у него есть решение в натуральных числах, то обязательно существует тройка натуральных чисел $p,q,k$ таких, что $p,q$ взаимно просты и разной четности и для этой тройки имеем два варианта -
первый вариант: система трех уравнений
$A^2-1=k(p^2-q^2), 6y^2=2kpq, 6Ay=k(p^2+q^2)$,
второй вариант: система трех уравнений
$A^2-1=2kpq, 6y^2=k(p^2-q^2), 6Ay=k(p^2+q^2)$
Решаем обе системы относительно переменных $A,y,k$.
Для обоих вариантов Maple дает:
первый вариант
$A=\dfrac{p^2+q^2}{\sqrt{p^4 - 12p^3{q} + 2p^2{q^2} + 12pq^3 + q^4}}$
$y=\dfrac{2pq}{\sqrt{p^4 - 12p^3{q} + 2p^2{q^2} + 12pq^3 + q^4}}$
$k=\dfrac{12pq}{p^4 - 12p^3{q} + 2p^2{q^2} + 12pq^3 + q^4}$
а также $A=\pm{1},k=0,y=0$
второй вариант
$A=\dfrac{p^2+q^2}{\sqrt{p^4 - 12p^3{q} + 2p^2{q^2} + 12pq^3 + q^4}}$
$y=\dfrac{p^2-q^2}{\sqrt{p^4 - 12p^3{q} + 2p^2{q^2} + 12pq^3 + q^4}}$
$k=\dfrac{6(p^2-q^2)}{p^4 - 12p^3{q} + 2p^2{q^2} + 12pq^3 + q^4}$
а также $A=\pm{1},k=0,y=0$.
Пусть необходимая тройка $p,q,k$ нашлась, тогда
$p^4 - 12p^3{q} + 2p^2{q^2} + 12pq^3 + q^4=d^2$, где $d$ натуральное число и
$A^2=\dfrac{(p^2+q^2)^2}{d^2}, y^2=\dfrac{4p^2{q^2}}{d^2}, k=\dfrac{12pq}{d^2}$ - первый вариант
$A^2=\dfrac{(p^2+q^2)^2}{d^2}, y^2=\dfrac{(p^2-q^2)^2}{d^2}, k=\dfrac{6(p^2-q^2)}{d^2}$ - второй вариант,
Поскольку в обоих вариантах числитель и знаменатель для натурального $A^2$ взаимно просты, что следует из $\gcd(p,q)=1$ и разной четности $p,q$, то $d^2=1$.
С помощью PARI/GP решаем уравнение Туэ $p^4 - 12p^3{q} + 2p^2{q^2} + 12pq^3 + q^4=1$
$p=\pm{1},q=0$ и $q=\pm{1},p=0$.
Необходимой тройки это не даёт, что и доказывает утверждение об отсутствии решения
исходного уравнения в натуральных числах при $R=3$.
Условие $k<1$ здесь не используется поскольку оно не верно.
Для первого варианта , например, $k>1$ при $(p,q)=(51,43),(223,188),(7384,6225)$
$k$ при этом, конечно, получаются дробные. То, что $k<1$ неверно, усложняет доказательство утверждения об отсутствии решения для произвольного натурального $R$.
Уравнения Туэ ,будут зависеть от параметра $R$. хотя для $R=4$ я уже проверил отсутствие решений $u,v$ для исходного уравнения. Рассмотрение других $R$ позже.

 Профиль  
                  
 
 Re: Задача с треугольными числами
Сообщение19.10.2023, 01:25 
Заслуженный участник
Аватара пользователя


21/11/12
1881
Санкт-Петербург
scwec в сообщении #1613831 писал(а):
получено уравнение $(A^2-1)^2+(6y^2)^2=(6y{A})^2\qquad(4)$
Не возникло бы у нас путаницы. Четверка задействована у меня выше, и это как раз то, что у Вас под радикалом. Значение этого полинома обозначено $\Delta_n,$ и далее
Andrey A в сообщении #1612931 писал(а):
$$d=\sqrt{\dfrac{q}{\Delta_n}}\ \ (5).$$ Чтобы радикал оказался рациональным числом, достаточно $$q\Delta_n=\square\ \ (6)$$
В случае $R=3/1$ знаменатель ($q,$ и тут тоже путаница) — единица, и конечно из $(5)$ следует $\Delta_n=1.$ Но для $R=\dfrac{3884}{3125}$ имеем $d=\sqrt{\dfrac{q}{\Delta_5}}=\sqrt{\dfrac{3125}{5}}=25.$ Пятерку Maple сама не найдет, ей надо задать. Однако, если она решает $\Delta_n=1$, то решит (в данном случае) и $ \Delta_n=5,\Delta_n=125,\Delta_n=3125.$ Дело сводится к факторизации знаменателя $R$, оно и есть общий случай. Или не решит?

 Профиль  
                  
 
 Re: Задача с треугольными числами
Сообщение19.10.2023, 12:02 
Заслуженный участник
Аватара пользователя


21/11/12
1881
Санкт-Петербург
P.S. Чтобы не усложнять, предлагаю следующее. Оставить всё же $R=\dfrac {p}{q};\  w=a^2+b^2,u=a^2-b^2,v=2ab;$ и переписать условие $\dfrac {w^2-1}{2uv}=R$ так: $$a^4 - 4 R a^3 b + 2 a^2 b^2 + 4 R a b^3 + b^4 = 1.$$ Если Maple решает такие Диофантовы уравнения о двух переменных, то и флаг ей в руки. Потребуются целые коэффициенты — распакуем $R;$ если ей захочется $\gcd (a,b)=1,$ факторизуем $q,$ ну и т.д. Короче, задача перестает быть неразрешимой.

 Профиль  
                  
 
 Re: Задача с треугольными числами
Сообщение19.10.2023, 14:22 
Заслуженный участник


17/09/10
2133
Andrey A в сообщении #1613881 писал(а):
Если Maple решает такие Диофантовы уравнения о двух переменных

Если речь идет об уравнении Туэ, то решает его PARI/GP и в данном случае $4R$ должно быть целым.
С решением тут всё не так просто. Позже выложу сообщение по поводу натуральных $R>3$.

 Профиль  
                  
Показать сообщения за:  Поле сортировки  
Начать новую тему Ответить на тему  [ Сообщений: 98 ]  На страницу Пред.  1, 2, 3, 4, 5 ... 7  След.

Модераторы: Модераторы Математики, Супермодераторы



Кто сейчас на конференции

Сейчас этот форум просматривают: нет зарегистрированных пользователей


Вы не можете начинать темы
Вы не можете отвечать на сообщения
Вы не можете редактировать свои сообщения
Вы не можете удалять свои сообщения
Вы не можете добавлять вложения

Найти:
Powered by phpBB © 2000, 2002, 2005, 2007 phpBB Group